SafeCracker #3 - We've Been Blocked [closed]What are the features of a well-written puzzle?The Guys - Logic #3 - The CasinoI've been everywhere, manCircuit DiagramA riddle that has been killing me the whole daySafeCracker #2 - The Mission Continues

Is there an strict difference between syntax and semantics?

Is there a push to use gender-neutral language and gender pronouns when given in the United States?

How honest to be with US immigration about uncertainty about travel plans?

Giving a character trauma but not "diagnosing" her?

Proofreading a novel: is it okay to use a question mark with an exclamation mark - "?!"

Is consistent disregard for students' time "normal" in undergraduate research?

Planet that goes through its entire history in a matter of Earth days

Can digital computers understand infinity?

SSD or HDD for server

What's the current zodiac?

An idiomatic word for "very little" in this context?

What is the type of this light bulb?

Skewer removal without quick release

What's the most efficient way to draw this region?

How can I cut a metal pipe while preserving the wires inside?

How should I tell a professor the answer to something he doesn't know?

Will the size of Bitcoin core full-node be too big to run on a normal computer?

Does the warlock's Gift of the Ever-Living Ones eldritch invocation work with potions or healing spells cast on you by others?

How to handle shared mortgage payment if one person can't pay their share?

Can you take an Immortal Phoenix out of the game?

What is the German word for: "It only works when I try to show you how it does not work"?

Is it realistic that an advanced species isn't good at war?

Raspberry Pi run commands on boot

How to make a PCB based on ATtiny easily updatable by end users?



SafeCracker #3 - We've Been Blocked [closed]


What are the features of a well-written puzzle?The Guys - Logic #3 - The CasinoI've been everywhere, manCircuit DiagramA riddle that has been killing me the whole daySafeCracker #2 - The Mission Continues






.everyoneloves__top-leaderboard:empty,.everyoneloves__mid-leaderboard:empty,.everyoneloves__bot-mid-leaderboard:empty
margin-bottom:0;

.everyonelovesstackoverflowposition:absolute;height:1px;width:1px;opacity:0;top:0;left:0;pointer-events:none;








-4














$begingroup$


SafeCracker 3enter image description here



Our Last Safe To Conquer



Ok, team, this one might give us a little trouble. Nothing we can't handle though... let's go...



This safe model is "The NevaCrak 9000" - the most secure safe there is.



Our only way in is to look at our puzzle, and simply find the block that doesn't belong.



A couple blocks are blank and are to be ignored. Only concern yourself with blocks with numbers and letters. Also disregard any of the flaire going on in the background.



THAT's IT



Everything is riding on you... go open that bad boy!




And here is the NevaCrak 9000




Which block does not belong??



( only one correct answer, no tricks or worplay involved )



The Safe










share|improve this question











$endgroup$






closed as too broad by noedne, PiIsNot3, Brandon_J, Peregrine Rook, Omega Krypton May 3 at 4:29


Please edit the question to limit it to a specific problem with enough detail to identify an adequate answer. Avoid asking multiple distinct questions at once. See the How to Ask page for help clarifying this question. If this question can be reworded to fit the rules in the help center, please edit the question.














  • 7




    $begingroup$
    @JohnS. "What comes next number puzzles" can also have many correct answers...
    $endgroup$
    – noedne
    May 2 at 17:25






  • 7




    $begingroup$
    @JohnS. This has nothing to do with being a new member but rather the content of the puzzle itself. We’re all trying to provide guidance on how you can improve your questions and what’s considered on-topic here. I sincerely hope you don’t take any of our criticism personally! :)
    $endgroup$
    – PiIsNot3
    May 2 at 23:32






  • 8




    $begingroup$
    @JohnS. I understand your frustration. You wrote a puzzle with a solution that clearly works when you find it. But I encourage you to think for a moment from the puzzle solver's perspective. Ak19 came up with a rule that works perfectly well to find an odd block out. From their perspective, they have no way of knowing that this is not the solution you intended. When you tell them it's not what you had in mind, it may feel unfair that their valid solution is deemed incorrect. It's important for puzzles to have unique solutions, but it's easy to miss things no matter how good your intentions.
    $endgroup$
    – noedne
    May 3 at 3:13






  • 5




    $begingroup$
    @JohnS. Compare the current question to a similar one: "What is the odd one out among the values 1, 2, 3, 4, 5?" Without any additional specifications from the author, any number of answers would be equally valid (5, because it's the only multiple of 5; 4, because it's the only composite number; etc. etc.), and thus the "correct" answer becomes a subjective choice. It's not the task itself that is problematic, but rather determining whether a possible answer is right or not. Also, I'm not sure how being a paid member of SE is relevant to this conversation.
    $endgroup$
    – PiIsNot3
    May 3 at 5:10







  • 7




    $begingroup$
    @JohnS. Please realize that many people enjoy asking their questions here not only because they like seeing how people approach their puzzles and like to see their efforts validated when people get pleasure from solving them, but also because this community is good about providing feedback on how to improve puzzlecrafting. Nothing in these comments is singling you or your puzzles out, and none of the advice or responses to you given by the community is in any way out of place. Please take the advice you've been offered in the spirit in which it was given; it may even improve your puzzles! :)
    $endgroup$
    – Rubio
    May 5 at 22:56

















-4














$begingroup$


SafeCracker 3enter image description here



Our Last Safe To Conquer



Ok, team, this one might give us a little trouble. Nothing we can't handle though... let's go...



This safe model is "The NevaCrak 9000" - the most secure safe there is.



Our only way in is to look at our puzzle, and simply find the block that doesn't belong.



A couple blocks are blank and are to be ignored. Only concern yourself with blocks with numbers and letters. Also disregard any of the flaire going on in the background.



THAT's IT



Everything is riding on you... go open that bad boy!




And here is the NevaCrak 9000




Which block does not belong??



( only one correct answer, no tricks or worplay involved )



The Safe










share|improve this question











$endgroup$






closed as too broad by noedne, PiIsNot3, Brandon_J, Peregrine Rook, Omega Krypton May 3 at 4:29


Please edit the question to limit it to a specific problem with enough detail to identify an adequate answer. Avoid asking multiple distinct questions at once. See the How to Ask page for help clarifying this question. If this question can be reworded to fit the rules in the help center, please edit the question.














  • 7




    $begingroup$
    @JohnS. "What comes next number puzzles" can also have many correct answers...
    $endgroup$
    – noedne
    May 2 at 17:25






  • 7




    $begingroup$
    @JohnS. This has nothing to do with being a new member but rather the content of the puzzle itself. We’re all trying to provide guidance on how you can improve your questions and what’s considered on-topic here. I sincerely hope you don’t take any of our criticism personally! :)
    $endgroup$
    – PiIsNot3
    May 2 at 23:32






  • 8




    $begingroup$
    @JohnS. I understand your frustration. You wrote a puzzle with a solution that clearly works when you find it. But I encourage you to think for a moment from the puzzle solver's perspective. Ak19 came up with a rule that works perfectly well to find an odd block out. From their perspective, they have no way of knowing that this is not the solution you intended. When you tell them it's not what you had in mind, it may feel unfair that their valid solution is deemed incorrect. It's important for puzzles to have unique solutions, but it's easy to miss things no matter how good your intentions.
    $endgroup$
    – noedne
    May 3 at 3:13






  • 5




    $begingroup$
    @JohnS. Compare the current question to a similar one: "What is the odd one out among the values 1, 2, 3, 4, 5?" Without any additional specifications from the author, any number of answers would be equally valid (5, because it's the only multiple of 5; 4, because it's the only composite number; etc. etc.), and thus the "correct" answer becomes a subjective choice. It's not the task itself that is problematic, but rather determining whether a possible answer is right or not. Also, I'm not sure how being a paid member of SE is relevant to this conversation.
    $endgroup$
    – PiIsNot3
    May 3 at 5:10







  • 7




    $begingroup$
    @JohnS. Please realize that many people enjoy asking their questions here not only because they like seeing how people approach their puzzles and like to see their efforts validated when people get pleasure from solving them, but also because this community is good about providing feedback on how to improve puzzlecrafting. Nothing in these comments is singling you or your puzzles out, and none of the advice or responses to you given by the community is in any way out of place. Please take the advice you've been offered in the spirit in which it was given; it may even improve your puzzles! :)
    $endgroup$
    – Rubio
    May 5 at 22:56













-4












-4








-4


1



$begingroup$


SafeCracker 3enter image description here



Our Last Safe To Conquer



Ok, team, this one might give us a little trouble. Nothing we can't handle though... let's go...



This safe model is "The NevaCrak 9000" - the most secure safe there is.



Our only way in is to look at our puzzle, and simply find the block that doesn't belong.



A couple blocks are blank and are to be ignored. Only concern yourself with blocks with numbers and letters. Also disregard any of the flaire going on in the background.



THAT's IT



Everything is riding on you... go open that bad boy!




And here is the NevaCrak 9000




Which block does not belong??



( only one correct answer, no tricks or worplay involved )



The Safe










share|improve this question











$endgroup$




SafeCracker 3enter image description here



Our Last Safe To Conquer



Ok, team, this one might give us a little trouble. Nothing we can't handle though... let's go...



This safe model is "The NevaCrak 9000" - the most secure safe there is.



Our only way in is to look at our puzzle, and simply find the block that doesn't belong.



A couple blocks are blank and are to be ignored. Only concern yourself with blocks with numbers and letters. Also disregard any of the flaire going on in the background.



THAT's IT



Everything is riding on you... go open that bad boy!




And here is the NevaCrak 9000




Which block does not belong??



( only one correct answer, no tricks or worplay involved )



The Safe







number-sequence formation-of-numbers grid-deduction algorithm algebra






share|improve this question















share|improve this question













share|improve this question




share|improve this question



share|improve this question








edited May 2 at 17:51







John S.

















asked May 2 at 15:29









John S.John S.

56612 bronze badges




56612 bronze badges





closed as too broad by noedne, PiIsNot3, Brandon_J, Peregrine Rook, Omega Krypton May 3 at 4:29


Please edit the question to limit it to a specific problem with enough detail to identify an adequate answer. Avoid asking multiple distinct questions at once. See the How to Ask page for help clarifying this question. If this question can be reworded to fit the rules in the help center, please edit the question.











closed as too broad by noedne, PiIsNot3, Brandon_J, Peregrine Rook, Omega Krypton May 3 at 4:29


Please edit the question to limit it to a specific problem with enough detail to identify an adequate answer. Avoid asking multiple distinct questions at once. See the How to Ask page for help clarifying this question. If this question can be reworded to fit the rules in the help center, please edit the question.









closed as too broad by noedne, PiIsNot3, Brandon_J, Peregrine Rook, Omega Krypton May 3 at 4:29


Please edit the question to limit it to a specific problem with enough detail to identify an adequate answer. Avoid asking multiple distinct questions at once. See the How to Ask page for help clarifying this question. If this question can be reworded to fit the rules in the help center, please edit the question.









  • 7




    $begingroup$
    @JohnS. "What comes next number puzzles" can also have many correct answers...
    $endgroup$
    – noedne
    May 2 at 17:25






  • 7




    $begingroup$
    @JohnS. This has nothing to do with being a new member but rather the content of the puzzle itself. We’re all trying to provide guidance on how you can improve your questions and what’s considered on-topic here. I sincerely hope you don’t take any of our criticism personally! :)
    $endgroup$
    – PiIsNot3
    May 2 at 23:32






  • 8




    $begingroup$
    @JohnS. I understand your frustration. You wrote a puzzle with a solution that clearly works when you find it. But I encourage you to think for a moment from the puzzle solver's perspective. Ak19 came up with a rule that works perfectly well to find an odd block out. From their perspective, they have no way of knowing that this is not the solution you intended. When you tell them it's not what you had in mind, it may feel unfair that their valid solution is deemed incorrect. It's important for puzzles to have unique solutions, but it's easy to miss things no matter how good your intentions.
    $endgroup$
    – noedne
    May 3 at 3:13






  • 5




    $begingroup$
    @JohnS. Compare the current question to a similar one: "What is the odd one out among the values 1, 2, 3, 4, 5?" Without any additional specifications from the author, any number of answers would be equally valid (5, because it's the only multiple of 5; 4, because it's the only composite number; etc. etc.), and thus the "correct" answer becomes a subjective choice. It's not the task itself that is problematic, but rather determining whether a possible answer is right or not. Also, I'm not sure how being a paid member of SE is relevant to this conversation.
    $endgroup$
    – PiIsNot3
    May 3 at 5:10







  • 7




    $begingroup$
    @JohnS. Please realize that many people enjoy asking their questions here not only because they like seeing how people approach their puzzles and like to see their efforts validated when people get pleasure from solving them, but also because this community is good about providing feedback on how to improve puzzlecrafting. Nothing in these comments is singling you or your puzzles out, and none of the advice or responses to you given by the community is in any way out of place. Please take the advice you've been offered in the spirit in which it was given; it may even improve your puzzles! :)
    $endgroup$
    – Rubio
    May 5 at 22:56












  • 7




    $begingroup$
    @JohnS. "What comes next number puzzles" can also have many correct answers...
    $endgroup$
    – noedne
    May 2 at 17:25






  • 7




    $begingroup$
    @JohnS. This has nothing to do with being a new member but rather the content of the puzzle itself. We’re all trying to provide guidance on how you can improve your questions and what’s considered on-topic here. I sincerely hope you don’t take any of our criticism personally! :)
    $endgroup$
    – PiIsNot3
    May 2 at 23:32






  • 8




    $begingroup$
    @JohnS. I understand your frustration. You wrote a puzzle with a solution that clearly works when you find it. But I encourage you to think for a moment from the puzzle solver's perspective. Ak19 came up with a rule that works perfectly well to find an odd block out. From their perspective, they have no way of knowing that this is not the solution you intended. When you tell them it's not what you had in mind, it may feel unfair that their valid solution is deemed incorrect. It's important for puzzles to have unique solutions, but it's easy to miss things no matter how good your intentions.
    $endgroup$
    – noedne
    May 3 at 3:13






  • 5




    $begingroup$
    @JohnS. Compare the current question to a similar one: "What is the odd one out among the values 1, 2, 3, 4, 5?" Without any additional specifications from the author, any number of answers would be equally valid (5, because it's the only multiple of 5; 4, because it's the only composite number; etc. etc.), and thus the "correct" answer becomes a subjective choice. It's not the task itself that is problematic, but rather determining whether a possible answer is right or not. Also, I'm not sure how being a paid member of SE is relevant to this conversation.
    $endgroup$
    – PiIsNot3
    May 3 at 5:10







  • 7




    $begingroup$
    @JohnS. Please realize that many people enjoy asking their questions here not only because they like seeing how people approach their puzzles and like to see their efforts validated when people get pleasure from solving them, but also because this community is good about providing feedback on how to improve puzzlecrafting. Nothing in these comments is singling you or your puzzles out, and none of the advice or responses to you given by the community is in any way out of place. Please take the advice you've been offered in the spirit in which it was given; it may even improve your puzzles! :)
    $endgroup$
    – Rubio
    May 5 at 22:56







7




7




$begingroup$
@JohnS. "What comes next number puzzles" can also have many correct answers...
$endgroup$
– noedne
May 2 at 17:25




$begingroup$
@JohnS. "What comes next number puzzles" can also have many correct answers...
$endgroup$
– noedne
May 2 at 17:25




7




7




$begingroup$
@JohnS. This has nothing to do with being a new member but rather the content of the puzzle itself. We’re all trying to provide guidance on how you can improve your questions and what’s considered on-topic here. I sincerely hope you don’t take any of our criticism personally! :)
$endgroup$
– PiIsNot3
May 2 at 23:32




$begingroup$
@JohnS. This has nothing to do with being a new member but rather the content of the puzzle itself. We’re all trying to provide guidance on how you can improve your questions and what’s considered on-topic here. I sincerely hope you don’t take any of our criticism personally! :)
$endgroup$
– PiIsNot3
May 2 at 23:32




8




8




$begingroup$
@JohnS. I understand your frustration. You wrote a puzzle with a solution that clearly works when you find it. But I encourage you to think for a moment from the puzzle solver's perspective. Ak19 came up with a rule that works perfectly well to find an odd block out. From their perspective, they have no way of knowing that this is not the solution you intended. When you tell them it's not what you had in mind, it may feel unfair that their valid solution is deemed incorrect. It's important for puzzles to have unique solutions, but it's easy to miss things no matter how good your intentions.
$endgroup$
– noedne
May 3 at 3:13




$begingroup$
@JohnS. I understand your frustration. You wrote a puzzle with a solution that clearly works when you find it. But I encourage you to think for a moment from the puzzle solver's perspective. Ak19 came up with a rule that works perfectly well to find an odd block out. From their perspective, they have no way of knowing that this is not the solution you intended. When you tell them it's not what you had in mind, it may feel unfair that their valid solution is deemed incorrect. It's important for puzzles to have unique solutions, but it's easy to miss things no matter how good your intentions.
$endgroup$
– noedne
May 3 at 3:13




5




5




$begingroup$
@JohnS. Compare the current question to a similar one: "What is the odd one out among the values 1, 2, 3, 4, 5?" Without any additional specifications from the author, any number of answers would be equally valid (5, because it's the only multiple of 5; 4, because it's the only composite number; etc. etc.), and thus the "correct" answer becomes a subjective choice. It's not the task itself that is problematic, but rather determining whether a possible answer is right or not. Also, I'm not sure how being a paid member of SE is relevant to this conversation.
$endgroup$
– PiIsNot3
May 3 at 5:10





$begingroup$
@JohnS. Compare the current question to a similar one: "What is the odd one out among the values 1, 2, 3, 4, 5?" Without any additional specifications from the author, any number of answers would be equally valid (5, because it's the only multiple of 5; 4, because it's the only composite number; etc. etc.), and thus the "correct" answer becomes a subjective choice. It's not the task itself that is problematic, but rather determining whether a possible answer is right or not. Also, I'm not sure how being a paid member of SE is relevant to this conversation.
$endgroup$
– PiIsNot3
May 3 at 5:10





7




7




$begingroup$
@JohnS. Please realize that many people enjoy asking their questions here not only because they like seeing how people approach their puzzles and like to see their efforts validated when people get pleasure from solving them, but also because this community is good about providing feedback on how to improve puzzlecrafting. Nothing in these comments is singling you or your puzzles out, and none of the advice or responses to you given by the community is in any way out of place. Please take the advice you've been offered in the spirit in which it was given; it may even improve your puzzles! :)
$endgroup$
– Rubio
May 5 at 22:56




$begingroup$
@JohnS. Please realize that many people enjoy asking their questions here not only because they like seeing how people approach their puzzles and like to see their efforts validated when people get pleasure from solving them, but also because this community is good about providing feedback on how to improve puzzlecrafting. Nothing in these comments is singling you or your puzzles out, and none of the advice or responses to you given by the community is in any way out of place. Please take the advice you've been offered in the spirit in which it was given; it may even improve your puzzles! :)
$endgroup$
– Rubio
May 5 at 22:56










10 Answers
10






active

oldest

votes


















7
















$begingroup$

The odd one out is the




green block with 5246.




This is because




for all the other numbers on the other blocks, combining the first and last digits and dividing by 2 gave the middle two digits. However, 5246 does not apply to that rule. To conform it to that rule, it would have to be 4248 (one of many possible fixes)...




Safe Cracked!






share|improve this answer












$endgroup$














  • $begingroup$
    The answer should be a single block (the ones that have a letter and number); not a complete group of blocks
    $endgroup$
    – Daniel Duque
    May 2 at 17:52










  • $begingroup$
    Yes. Ok, I am giving a very small , yet critical Hint: It's all a numbers game, it's allll a numbers game.
    $endgroup$
    – John S.
    May 2 at 18:32






  • 2




    $begingroup$
    @JohnS. Didn't Ak19 correctly identify this block in an earlier answer?
    $endgroup$
    – noedne
    May 2 at 23:40






  • 3




    $begingroup$
    @JohnS. While Invent's original answer (divisible by 3) is an accurate pattern and a good find, Ak19 posted it first. It's also not the same as your "correct" answer. At all. You just got tired of waiting for someone to discover your solution and decided to give it away to someone.
    $endgroup$
    – David K
    May 3 at 12:12






  • 2




    $begingroup$
    No, there are many correct answers, as I have been pointed out. I wlll never make a puzzle with multiple solutions again. Not intentional anyways. He got a correct answer, we can't take that away from him. I am new here, so this is a learning lesson. I will be more specific in the future if I submit more. Thank you for your comment, though.
    $endgroup$
    – John S.
    May 3 at 18:28


















10
















$begingroup$

The




green block with the number 5246




doesn't belong, because




it is the only number not divisible by 3 where all the others are.







share|improve this answer












$endgroup$














  • $begingroup$
    Ak19, that is a very good suggested answer, however, it is not correct. You are thinking along the correct path, however.
    $endgroup$
    – John S.
    May 2 at 16:48










  • $begingroup$
    @Invent: 99.99999999%
    $endgroup$
    – John S.
    May 2 at 17:55






  • 2




    $begingroup$
    This is the right answer, for a slightly weaker reason than the ‘‘official’’ one.
    $endgroup$
    – Peregrine Rook
    May 3 at 2:20






  • 1




    $begingroup$
    all the numbers had exactly 3 prime factors. they were all (2,3,X) for mostly very large X (ranging from 11 to 1583). The only outlier is 5246 with prime factors of (2,43,61) once I saw this I couldn't look away!
    $endgroup$
    – stew
    May 3 at 7:13






  • 1




    $begingroup$
    @stew: Your comment is essentially the same as David K’s answer.
    $endgroup$
    – Peregrine Rook
    May 4 at 16:17


















3
















$begingroup$

I'm going to guess the:




Black block.

It's the only one that isn't a tetris piece :P







share|improve this answer










$endgroup$






















    3
















    $begingroup$

    I will guess that it is




    Green block 5246




    My reasoning is that




    All other blocks have numbers that are divisible by 6, and it is the only block that has two prime factor other than 2 and 3. The prime factors of all other numbers are some number of 2s and 3s, and one other larger prime number. 5246 factorizes to 2, 43, and 61.







    share|improve this answer










    $endgroup$










    • 1




      $begingroup$
      This is equivalent to Ak19’s answer, but worded in a more complicated way.
      $endgroup$
      – Peregrine Rook
      May 3 at 2:21






    • 1




      $begingroup$
      @PeregrineRook It is an expansion of Ak19's answer, but they are not equivalent. The OP indicated that Ak19 was thinking along the correct path, so I dug deeper in that direction.
      $endgroup$
      – David K
      May 3 at 11:53






    • 2




      $begingroup$
      Ah, I see.  OK, you’re right.  Your answer encompasses Ak19’s answer, but you found something more. (Now it turns out that Ak19’s answer was very close to what the OP was looking for, and yours was not, but that doesn’t reflect badly on your answer.  It just goes to support the argument that the question is too broad, because it has too many different answers.) I would upvote you, but I already did.
      $endgroup$
      – Peregrine Rook
      May 4 at 16:17


















    3
















    $begingroup$

    The answer is:




    The green block with the number 9498.




    because




    The average of the numbers on the green blocks is not a whole, even number (as it is for the other sets of blocks) and the number 9498 here is a duplicate of another block. This block could be 9496 which would make it valid.







    share|improve this answer












    $endgroup$






















      3
















      $begingroup$

      My answer:




      When arranging the numbers in the order of the word Stack Exchange, and then doing an absolute value of the difference between any number and the following number, I get a pattern that the difference value rises then drops alternatingly, except at the Black A where the value remains the same. (Differences: 4176,8214,8214,3150,5088,1950,5274,2152,4252,0,4224,3126 - rise fall neutral fall rise fall rise fall rise fall rise fall)




      Below is Meant as comment, but alas



      @John S posted the following in a comment at https://puzzling.stackexchange.com/a/83526/60039




      [ C, 8412, E, 174,H, 5246,C, 7398,A, 9498,S, 4236,A, 198,T, 8412,N, 9498,G, 5274,E, 2148 ]




      It is missing data.




      The last three are the orange blocks, the second to last three are the black blocks, the third to last three are the green blocks and the first 2 are only half of the yellow blocks, the x and k are missing.




      However:




      There are two pairs letters with same numbers, 9498 and 8412.




      Also:




      John stated elsewhere its just a numbers game...which makes sense because the letters spell out Stack Exchange







      share|improve this answer












      $endgroup$






















        2
















        $begingroup$

        I would guess it is the:




        Yellow block that has the E and the number 174




        Why?




        All other group of blocks follow the pattern of increasing/decreasing the number as the letters advance through the alphabet.







        share|improve this answer










        $endgroup$






















          2
















          $begingroup$

          Answer:




          The Black A




          Reason:




          It's the only one that has 11 as a factor







          share|improve this answer










          $endgroup$






















            1
















            $begingroup$

            The answer might be:




            The yellow one




            Reasoning:




            All the other blocks have only letters when looked at from a specific direction.







            share|improve this answer










            $endgroup$














            • $begingroup$
              No, sorry. We are looking for an individual block that just doesn't fit the criteria of all the others. - But that was a reasonable guess!
              $endgroup$
              – John S.
              May 2 at 15:40











            • $begingroup$
              HERE IS THE DATA SET FOR THIS PUZZLE... Now, we are back to our regularly scheduled "What doesn't belong in this pattern?" puzzle. [ C, 8412, E, 174,H, 5246,C, 7398,A, 9498,S, 4236,A, 198,T, 8412,N, 9498,G, 5274,E, 2148 ]
              $endgroup$
              – John S.
              May 2 at 19:06











            • $begingroup$
              Delete this question. Not editing it. ANSWER WAS SIMPLE: take first number and last number , combine. Divide by 2. will give you middle number. ONLY ONE block doesn't fit criteria. People had fun trying, you can delete. Thanks.
              $endgroup$
              – John S.
              May 3 at 11:20


















            1
















            $begingroup$

            I noticed that




            if you open out the net of each, centred on the side showing the letter, any numbers above, below and to the right are all aligned in the same direction as the letter; and the number on the left is on its side, with its top pointing to the left. The only exception is the Yellow K block, where the number on the right isn't aligned in the same direction as the letter.







            share|improve this answer










            $endgroup$










            • 1




              $begingroup$
              Looks like there is more than one valid solution :(
              $endgroup$
              – MichaelMaggs
              May 2 at 16:51






            • 5




              $begingroup$
              If there's more than one block that's different from the others in a notable way and the challenge is to figure out what specific difference you had in mind, then the puzzle is basically "read the puzzle author's mind". Let's hope it turns out that the odd-one-out property John S intends is much more compelling than the two other perfectly plausible ones suggested so far...
              $endgroup$
              – Gareth McCaughan
              May 2 at 17:42






            • 1




              $begingroup$
              It is the other way around >! This is a hint.
              $endgroup$
              – Daniel Duque
              May 2 at 18:33






            • 1




              $begingroup$
              @JohnS. typically we use Rot13 to hide hints or spoilers in comments. But I’d strongly recommend you put the hints in the original post, since they would be considered part of the overall puzzle
              $endgroup$
              – PiIsNot3
              May 2 at 19:12







            • 1




              $begingroup$
              @JohnS. A "What's next?" like you describe should also be closed for being too broad...
              $endgroup$
              – noedne
              May 2 at 21:24


















            10 Answers
            10






            active

            oldest

            votes








            10 Answers
            10






            active

            oldest

            votes









            active

            oldest

            votes






            active

            oldest

            votes









            7
















            $begingroup$

            The odd one out is the




            green block with 5246.




            This is because




            for all the other numbers on the other blocks, combining the first and last digits and dividing by 2 gave the middle two digits. However, 5246 does not apply to that rule. To conform it to that rule, it would have to be 4248 (one of many possible fixes)...




            Safe Cracked!






            share|improve this answer












            $endgroup$














            • $begingroup$
              The answer should be a single block (the ones that have a letter and number); not a complete group of blocks
              $endgroup$
              – Daniel Duque
              May 2 at 17:52










            • $begingroup$
              Yes. Ok, I am giving a very small , yet critical Hint: It's all a numbers game, it's allll a numbers game.
              $endgroup$
              – John S.
              May 2 at 18:32






            • 2




              $begingroup$
              @JohnS. Didn't Ak19 correctly identify this block in an earlier answer?
              $endgroup$
              – noedne
              May 2 at 23:40






            • 3




              $begingroup$
              @JohnS. While Invent's original answer (divisible by 3) is an accurate pattern and a good find, Ak19 posted it first. It's also not the same as your "correct" answer. At all. You just got tired of waiting for someone to discover your solution and decided to give it away to someone.
              $endgroup$
              – David K
              May 3 at 12:12






            • 2




              $begingroup$
              No, there are many correct answers, as I have been pointed out. I wlll never make a puzzle with multiple solutions again. Not intentional anyways. He got a correct answer, we can't take that away from him. I am new here, so this is a learning lesson. I will be more specific in the future if I submit more. Thank you for your comment, though.
              $endgroup$
              – John S.
              May 3 at 18:28















            7
















            $begingroup$

            The odd one out is the




            green block with 5246.




            This is because




            for all the other numbers on the other blocks, combining the first and last digits and dividing by 2 gave the middle two digits. However, 5246 does not apply to that rule. To conform it to that rule, it would have to be 4248 (one of many possible fixes)...




            Safe Cracked!






            share|improve this answer












            $endgroup$














            • $begingroup$
              The answer should be a single block (the ones that have a letter and number); not a complete group of blocks
              $endgroup$
              – Daniel Duque
              May 2 at 17:52










            • $begingroup$
              Yes. Ok, I am giving a very small , yet critical Hint: It's all a numbers game, it's allll a numbers game.
              $endgroup$
              – John S.
              May 2 at 18:32






            • 2




              $begingroup$
              @JohnS. Didn't Ak19 correctly identify this block in an earlier answer?
              $endgroup$
              – noedne
              May 2 at 23:40






            • 3




              $begingroup$
              @JohnS. While Invent's original answer (divisible by 3) is an accurate pattern and a good find, Ak19 posted it first. It's also not the same as your "correct" answer. At all. You just got tired of waiting for someone to discover your solution and decided to give it away to someone.
              $endgroup$
              – David K
              May 3 at 12:12






            • 2




              $begingroup$
              No, there are many correct answers, as I have been pointed out. I wlll never make a puzzle with multiple solutions again. Not intentional anyways. He got a correct answer, we can't take that away from him. I am new here, so this is a learning lesson. I will be more specific in the future if I submit more. Thank you for your comment, though.
              $endgroup$
              – John S.
              May 3 at 18:28













            7














            7










            7







            $begingroup$

            The odd one out is the




            green block with 5246.




            This is because




            for all the other numbers on the other blocks, combining the first and last digits and dividing by 2 gave the middle two digits. However, 5246 does not apply to that rule. To conform it to that rule, it would have to be 4248 (one of many possible fixes)...




            Safe Cracked!






            share|improve this answer












            $endgroup$



            The odd one out is the




            green block with 5246.




            This is because




            for all the other numbers on the other blocks, combining the first and last digits and dividing by 2 gave the middle two digits. However, 5246 does not apply to that rule. To conform it to that rule, it would have to be 4248 (one of many possible fixes)...




            Safe Cracked!







            share|improve this answer















            share|improve this answer




            share|improve this answer



            share|improve this answer








            edited May 2 at 23:24

























            answered May 2 at 16:47









            Voldemort's WrathVoldemort's Wrath

            4471 silver badge23 bronze badges




            4471 silver badge23 bronze badges














            • $begingroup$
              The answer should be a single block (the ones that have a letter and number); not a complete group of blocks
              $endgroup$
              – Daniel Duque
              May 2 at 17:52










            • $begingroup$
              Yes. Ok, I am giving a very small , yet critical Hint: It's all a numbers game, it's allll a numbers game.
              $endgroup$
              – John S.
              May 2 at 18:32






            • 2




              $begingroup$
              @JohnS. Didn't Ak19 correctly identify this block in an earlier answer?
              $endgroup$
              – noedne
              May 2 at 23:40






            • 3




              $begingroup$
              @JohnS. While Invent's original answer (divisible by 3) is an accurate pattern and a good find, Ak19 posted it first. It's also not the same as your "correct" answer. At all. You just got tired of waiting for someone to discover your solution and decided to give it away to someone.
              $endgroup$
              – David K
              May 3 at 12:12






            • 2




              $begingroup$
              No, there are many correct answers, as I have been pointed out. I wlll never make a puzzle with multiple solutions again. Not intentional anyways. He got a correct answer, we can't take that away from him. I am new here, so this is a learning lesson. I will be more specific in the future if I submit more. Thank you for your comment, though.
              $endgroup$
              – John S.
              May 3 at 18:28
















            • $begingroup$
              The answer should be a single block (the ones that have a letter and number); not a complete group of blocks
              $endgroup$
              – Daniel Duque
              May 2 at 17:52










            • $begingroup$
              Yes. Ok, I am giving a very small , yet critical Hint: It's all a numbers game, it's allll a numbers game.
              $endgroup$
              – John S.
              May 2 at 18:32






            • 2




              $begingroup$
              @JohnS. Didn't Ak19 correctly identify this block in an earlier answer?
              $endgroup$
              – noedne
              May 2 at 23:40






            • 3




              $begingroup$
              @JohnS. While Invent's original answer (divisible by 3) is an accurate pattern and a good find, Ak19 posted it first. It's also not the same as your "correct" answer. At all. You just got tired of waiting for someone to discover your solution and decided to give it away to someone.
              $endgroup$
              – David K
              May 3 at 12:12






            • 2




              $begingroup$
              No, there are many correct answers, as I have been pointed out. I wlll never make a puzzle with multiple solutions again. Not intentional anyways. He got a correct answer, we can't take that away from him. I am new here, so this is a learning lesson. I will be more specific in the future if I submit more. Thank you for your comment, though.
              $endgroup$
              – John S.
              May 3 at 18:28















            $begingroup$
            The answer should be a single block (the ones that have a letter and number); not a complete group of blocks
            $endgroup$
            – Daniel Duque
            May 2 at 17:52




            $begingroup$
            The answer should be a single block (the ones that have a letter and number); not a complete group of blocks
            $endgroup$
            – Daniel Duque
            May 2 at 17:52












            $begingroup$
            Yes. Ok, I am giving a very small , yet critical Hint: It's all a numbers game, it's allll a numbers game.
            $endgroup$
            – John S.
            May 2 at 18:32




            $begingroup$
            Yes. Ok, I am giving a very small , yet critical Hint: It's all a numbers game, it's allll a numbers game.
            $endgroup$
            – John S.
            May 2 at 18:32




            2




            2




            $begingroup$
            @JohnS. Didn't Ak19 correctly identify this block in an earlier answer?
            $endgroup$
            – noedne
            May 2 at 23:40




            $begingroup$
            @JohnS. Didn't Ak19 correctly identify this block in an earlier answer?
            $endgroup$
            – noedne
            May 2 at 23:40




            3




            3




            $begingroup$
            @JohnS. While Invent's original answer (divisible by 3) is an accurate pattern and a good find, Ak19 posted it first. It's also not the same as your "correct" answer. At all. You just got tired of waiting for someone to discover your solution and decided to give it away to someone.
            $endgroup$
            – David K
            May 3 at 12:12




            $begingroup$
            @JohnS. While Invent's original answer (divisible by 3) is an accurate pattern and a good find, Ak19 posted it first. It's also not the same as your "correct" answer. At all. You just got tired of waiting for someone to discover your solution and decided to give it away to someone.
            $endgroup$
            – David K
            May 3 at 12:12




            2




            2




            $begingroup$
            No, there are many correct answers, as I have been pointed out. I wlll never make a puzzle with multiple solutions again. Not intentional anyways. He got a correct answer, we can't take that away from him. I am new here, so this is a learning lesson. I will be more specific in the future if I submit more. Thank you for your comment, though.
            $endgroup$
            – John S.
            May 3 at 18:28




            $begingroup$
            No, there are many correct answers, as I have been pointed out. I wlll never make a puzzle with multiple solutions again. Not intentional anyways. He got a correct answer, we can't take that away from him. I am new here, so this is a learning lesson. I will be more specific in the future if I submit more. Thank you for your comment, though.
            $endgroup$
            – John S.
            May 3 at 18:28













            10
















            $begingroup$

            The




            green block with the number 5246




            doesn't belong, because




            it is the only number not divisible by 3 where all the others are.







            share|improve this answer












            $endgroup$














            • $begingroup$
              Ak19, that is a very good suggested answer, however, it is not correct. You are thinking along the correct path, however.
              $endgroup$
              – John S.
              May 2 at 16:48










            • $begingroup$
              @Invent: 99.99999999%
              $endgroup$
              – John S.
              May 2 at 17:55






            • 2




              $begingroup$
              This is the right answer, for a slightly weaker reason than the ‘‘official’’ one.
              $endgroup$
              – Peregrine Rook
              May 3 at 2:20






            • 1




              $begingroup$
              all the numbers had exactly 3 prime factors. they were all (2,3,X) for mostly very large X (ranging from 11 to 1583). The only outlier is 5246 with prime factors of (2,43,61) once I saw this I couldn't look away!
              $endgroup$
              – stew
              May 3 at 7:13






            • 1




              $begingroup$
              @stew: Your comment is essentially the same as David K’s answer.
              $endgroup$
              – Peregrine Rook
              May 4 at 16:17















            10
















            $begingroup$

            The




            green block with the number 5246




            doesn't belong, because




            it is the only number not divisible by 3 where all the others are.







            share|improve this answer












            $endgroup$














            • $begingroup$
              Ak19, that is a very good suggested answer, however, it is not correct. You are thinking along the correct path, however.
              $endgroup$
              – John S.
              May 2 at 16:48










            • $begingroup$
              @Invent: 99.99999999%
              $endgroup$
              – John S.
              May 2 at 17:55






            • 2




              $begingroup$
              This is the right answer, for a slightly weaker reason than the ‘‘official’’ one.
              $endgroup$
              – Peregrine Rook
              May 3 at 2:20






            • 1




              $begingroup$
              all the numbers had exactly 3 prime factors. they were all (2,3,X) for mostly very large X (ranging from 11 to 1583). The only outlier is 5246 with prime factors of (2,43,61) once I saw this I couldn't look away!
              $endgroup$
              – stew
              May 3 at 7:13






            • 1




              $begingroup$
              @stew: Your comment is essentially the same as David K’s answer.
              $endgroup$
              – Peregrine Rook
              May 4 at 16:17













            10














            10










            10







            $begingroup$

            The




            green block with the number 5246




            doesn't belong, because




            it is the only number not divisible by 3 where all the others are.







            share|improve this answer












            $endgroup$



            The




            green block with the number 5246




            doesn't belong, because




            it is the only number not divisible by 3 where all the others are.








            share|improve this answer















            share|improve this answer




            share|improve this answer



            share|improve this answer








            edited May 2 at 16:44









            Voldemort's Wrath

            4471 silver badge23 bronze badges




            4471 silver badge23 bronze badges










            answered May 2 at 16:41









            Ak19Ak19

            1,9881 gold badge3 silver badges30 bronze badges




            1,9881 gold badge3 silver badges30 bronze badges














            • $begingroup$
              Ak19, that is a very good suggested answer, however, it is not correct. You are thinking along the correct path, however.
              $endgroup$
              – John S.
              May 2 at 16:48










            • $begingroup$
              @Invent: 99.99999999%
              $endgroup$
              – John S.
              May 2 at 17:55






            • 2




              $begingroup$
              This is the right answer, for a slightly weaker reason than the ‘‘official’’ one.
              $endgroup$
              – Peregrine Rook
              May 3 at 2:20






            • 1




              $begingroup$
              all the numbers had exactly 3 prime factors. they were all (2,3,X) for mostly very large X (ranging from 11 to 1583). The only outlier is 5246 with prime factors of (2,43,61) once I saw this I couldn't look away!
              $endgroup$
              – stew
              May 3 at 7:13






            • 1




              $begingroup$
              @stew: Your comment is essentially the same as David K’s answer.
              $endgroup$
              – Peregrine Rook
              May 4 at 16:17
















            • $begingroup$
              Ak19, that is a very good suggested answer, however, it is not correct. You are thinking along the correct path, however.
              $endgroup$
              – John S.
              May 2 at 16:48










            • $begingroup$
              @Invent: 99.99999999%
              $endgroup$
              – John S.
              May 2 at 17:55






            • 2




              $begingroup$
              This is the right answer, for a slightly weaker reason than the ‘‘official’’ one.
              $endgroup$
              – Peregrine Rook
              May 3 at 2:20






            • 1




              $begingroup$
              all the numbers had exactly 3 prime factors. they were all (2,3,X) for mostly very large X (ranging from 11 to 1583). The only outlier is 5246 with prime factors of (2,43,61) once I saw this I couldn't look away!
              $endgroup$
              – stew
              May 3 at 7:13






            • 1




              $begingroup$
              @stew: Your comment is essentially the same as David K’s answer.
              $endgroup$
              – Peregrine Rook
              May 4 at 16:17















            $begingroup$
            Ak19, that is a very good suggested answer, however, it is not correct. You are thinking along the correct path, however.
            $endgroup$
            – John S.
            May 2 at 16:48




            $begingroup$
            Ak19, that is a very good suggested answer, however, it is not correct. You are thinking along the correct path, however.
            $endgroup$
            – John S.
            May 2 at 16:48












            $begingroup$
            @Invent: 99.99999999%
            $endgroup$
            – John S.
            May 2 at 17:55




            $begingroup$
            @Invent: 99.99999999%
            $endgroup$
            – John S.
            May 2 at 17:55




            2




            2




            $begingroup$
            This is the right answer, for a slightly weaker reason than the ‘‘official’’ one.
            $endgroup$
            – Peregrine Rook
            May 3 at 2:20




            $begingroup$
            This is the right answer, for a slightly weaker reason than the ‘‘official’’ one.
            $endgroup$
            – Peregrine Rook
            May 3 at 2:20




            1




            1




            $begingroup$
            all the numbers had exactly 3 prime factors. they were all (2,3,X) for mostly very large X (ranging from 11 to 1583). The only outlier is 5246 with prime factors of (2,43,61) once I saw this I couldn't look away!
            $endgroup$
            – stew
            May 3 at 7:13




            $begingroup$
            all the numbers had exactly 3 prime factors. they were all (2,3,X) for mostly very large X (ranging from 11 to 1583). The only outlier is 5246 with prime factors of (2,43,61) once I saw this I couldn't look away!
            $endgroup$
            – stew
            May 3 at 7:13




            1




            1




            $begingroup$
            @stew: Your comment is essentially the same as David K’s answer.
            $endgroup$
            – Peregrine Rook
            May 4 at 16:17




            $begingroup$
            @stew: Your comment is essentially the same as David K’s answer.
            $endgroup$
            – Peregrine Rook
            May 4 at 16:17











            3
















            $begingroup$

            I'm going to guess the:




            Black block.

            It's the only one that isn't a tetris piece :P







            share|improve this answer










            $endgroup$



















              3
















              $begingroup$

              I'm going to guess the:




              Black block.

              It's the only one that isn't a tetris piece :P







              share|improve this answer










              $endgroup$

















                3














                3










                3







                $begingroup$

                I'm going to guess the:




                Black block.

                It's the only one that isn't a tetris piece :P







                share|improve this answer










                $endgroup$



                I'm going to guess the:




                Black block.

                It's the only one that isn't a tetris piece :P








                share|improve this answer













                share|improve this answer




                share|improve this answer



                share|improve this answer










                answered May 2 at 16:09









                SmockSmock

                80610 bronze badges




                80610 bronze badges
























                    3
















                    $begingroup$

                    I will guess that it is




                    Green block 5246




                    My reasoning is that




                    All other blocks have numbers that are divisible by 6, and it is the only block that has two prime factor other than 2 and 3. The prime factors of all other numbers are some number of 2s and 3s, and one other larger prime number. 5246 factorizes to 2, 43, and 61.







                    share|improve this answer










                    $endgroup$










                    • 1




                      $begingroup$
                      This is equivalent to Ak19’s answer, but worded in a more complicated way.
                      $endgroup$
                      – Peregrine Rook
                      May 3 at 2:21






                    • 1




                      $begingroup$
                      @PeregrineRook It is an expansion of Ak19's answer, but they are not equivalent. The OP indicated that Ak19 was thinking along the correct path, so I dug deeper in that direction.
                      $endgroup$
                      – David K
                      May 3 at 11:53






                    • 2




                      $begingroup$
                      Ah, I see.  OK, you’re right.  Your answer encompasses Ak19’s answer, but you found something more. (Now it turns out that Ak19’s answer was very close to what the OP was looking for, and yours was not, but that doesn’t reflect badly on your answer.  It just goes to support the argument that the question is too broad, because it has too many different answers.) I would upvote you, but I already did.
                      $endgroup$
                      – Peregrine Rook
                      May 4 at 16:17















                    3
















                    $begingroup$

                    I will guess that it is




                    Green block 5246




                    My reasoning is that




                    All other blocks have numbers that are divisible by 6, and it is the only block that has two prime factor other than 2 and 3. The prime factors of all other numbers are some number of 2s and 3s, and one other larger prime number. 5246 factorizes to 2, 43, and 61.







                    share|improve this answer










                    $endgroup$










                    • 1




                      $begingroup$
                      This is equivalent to Ak19’s answer, but worded in a more complicated way.
                      $endgroup$
                      – Peregrine Rook
                      May 3 at 2:21






                    • 1




                      $begingroup$
                      @PeregrineRook It is an expansion of Ak19's answer, but they are not equivalent. The OP indicated that Ak19 was thinking along the correct path, so I dug deeper in that direction.
                      $endgroup$
                      – David K
                      May 3 at 11:53






                    • 2




                      $begingroup$
                      Ah, I see.  OK, you’re right.  Your answer encompasses Ak19’s answer, but you found something more. (Now it turns out that Ak19’s answer was very close to what the OP was looking for, and yours was not, but that doesn’t reflect badly on your answer.  It just goes to support the argument that the question is too broad, because it has too many different answers.) I would upvote you, but I already did.
                      $endgroup$
                      – Peregrine Rook
                      May 4 at 16:17













                    3














                    3










                    3







                    $begingroup$

                    I will guess that it is




                    Green block 5246




                    My reasoning is that




                    All other blocks have numbers that are divisible by 6, and it is the only block that has two prime factor other than 2 and 3. The prime factors of all other numbers are some number of 2s and 3s, and one other larger prime number. 5246 factorizes to 2, 43, and 61.







                    share|improve this answer










                    $endgroup$



                    I will guess that it is




                    Green block 5246




                    My reasoning is that




                    All other blocks have numbers that are divisible by 6, and it is the only block that has two prime factor other than 2 and 3. The prime factors of all other numbers are some number of 2s and 3s, and one other larger prime number. 5246 factorizes to 2, 43, and 61.








                    share|improve this answer













                    share|improve this answer




                    share|improve this answer



                    share|improve this answer










                    answered May 2 at 20:18









                    David KDavid K

                    2451 silver badge9 bronze badges




                    2451 silver badge9 bronze badges










                    • 1




                      $begingroup$
                      This is equivalent to Ak19’s answer, but worded in a more complicated way.
                      $endgroup$
                      – Peregrine Rook
                      May 3 at 2:21






                    • 1




                      $begingroup$
                      @PeregrineRook It is an expansion of Ak19's answer, but they are not equivalent. The OP indicated that Ak19 was thinking along the correct path, so I dug deeper in that direction.
                      $endgroup$
                      – David K
                      May 3 at 11:53






                    • 2




                      $begingroup$
                      Ah, I see.  OK, you’re right.  Your answer encompasses Ak19’s answer, but you found something more. (Now it turns out that Ak19’s answer was very close to what the OP was looking for, and yours was not, but that doesn’t reflect badly on your answer.  It just goes to support the argument that the question is too broad, because it has too many different answers.) I would upvote you, but I already did.
                      $endgroup$
                      – Peregrine Rook
                      May 4 at 16:17












                    • 1




                      $begingroup$
                      This is equivalent to Ak19’s answer, but worded in a more complicated way.
                      $endgroup$
                      – Peregrine Rook
                      May 3 at 2:21






                    • 1




                      $begingroup$
                      @PeregrineRook It is an expansion of Ak19's answer, but they are not equivalent. The OP indicated that Ak19 was thinking along the correct path, so I dug deeper in that direction.
                      $endgroup$
                      – David K
                      May 3 at 11:53






                    • 2




                      $begingroup$
                      Ah, I see.  OK, you’re right.  Your answer encompasses Ak19’s answer, but you found something more. (Now it turns out that Ak19’s answer was very close to what the OP was looking for, and yours was not, but that doesn’t reflect badly on your answer.  It just goes to support the argument that the question is too broad, because it has too many different answers.) I would upvote you, but I already did.
                      $endgroup$
                      – Peregrine Rook
                      May 4 at 16:17







                    1




                    1




                    $begingroup$
                    This is equivalent to Ak19’s answer, but worded in a more complicated way.
                    $endgroup$
                    – Peregrine Rook
                    May 3 at 2:21




                    $begingroup$
                    This is equivalent to Ak19’s answer, but worded in a more complicated way.
                    $endgroup$
                    – Peregrine Rook
                    May 3 at 2:21




                    1




                    1




                    $begingroup$
                    @PeregrineRook It is an expansion of Ak19's answer, but they are not equivalent. The OP indicated that Ak19 was thinking along the correct path, so I dug deeper in that direction.
                    $endgroup$
                    – David K
                    May 3 at 11:53




                    $begingroup$
                    @PeregrineRook It is an expansion of Ak19's answer, but they are not equivalent. The OP indicated that Ak19 was thinking along the correct path, so I dug deeper in that direction.
                    $endgroup$
                    – David K
                    May 3 at 11:53




                    2




                    2




                    $begingroup$
                    Ah, I see.  OK, you’re right.  Your answer encompasses Ak19’s answer, but you found something more. (Now it turns out that Ak19’s answer was very close to what the OP was looking for, and yours was not, but that doesn’t reflect badly on your answer.  It just goes to support the argument that the question is too broad, because it has too many different answers.) I would upvote you, but I already did.
                    $endgroup$
                    – Peregrine Rook
                    May 4 at 16:17




                    $begingroup$
                    Ah, I see.  OK, you’re right.  Your answer encompasses Ak19’s answer, but you found something more. (Now it turns out that Ak19’s answer was very close to what the OP was looking for, and yours was not, but that doesn’t reflect badly on your answer.  It just goes to support the argument that the question is too broad, because it has too many different answers.) I would upvote you, but I already did.
                    $endgroup$
                    – Peregrine Rook
                    May 4 at 16:17











                    3
















                    $begingroup$

                    The answer is:




                    The green block with the number 9498.




                    because




                    The average of the numbers on the green blocks is not a whole, even number (as it is for the other sets of blocks) and the number 9498 here is a duplicate of another block. This block could be 9496 which would make it valid.







                    share|improve this answer












                    $endgroup$



















                      3
















                      $begingroup$

                      The answer is:




                      The green block with the number 9498.




                      because




                      The average of the numbers on the green blocks is not a whole, even number (as it is for the other sets of blocks) and the number 9498 here is a duplicate of another block. This block could be 9496 which would make it valid.







                      share|improve this answer












                      $endgroup$

















                        3














                        3










                        3







                        $begingroup$

                        The answer is:




                        The green block with the number 9498.




                        because




                        The average of the numbers on the green blocks is not a whole, even number (as it is for the other sets of blocks) and the number 9498 here is a duplicate of another block. This block could be 9496 which would make it valid.







                        share|improve this answer












                        $endgroup$



                        The answer is:




                        The green block with the number 9498.




                        because




                        The average of the numbers on the green blocks is not a whole, even number (as it is for the other sets of blocks) and the number 9498 here is a duplicate of another block. This block could be 9496 which would make it valid.








                        share|improve this answer















                        share|improve this answer




                        share|improve this answer



                        share|improve this answer








                        edited May 2 at 20:36

























                        answered May 2 at 20:30









                        Amru E.Amru E.

                        1313 bronze badges




                        1313 bronze badges
























                            3
















                            $begingroup$

                            My answer:




                            When arranging the numbers in the order of the word Stack Exchange, and then doing an absolute value of the difference between any number and the following number, I get a pattern that the difference value rises then drops alternatingly, except at the Black A where the value remains the same. (Differences: 4176,8214,8214,3150,5088,1950,5274,2152,4252,0,4224,3126 - rise fall neutral fall rise fall rise fall rise fall rise fall)




                            Below is Meant as comment, but alas



                            @John S posted the following in a comment at https://puzzling.stackexchange.com/a/83526/60039




                            [ C, 8412, E, 174,H, 5246,C, 7398,A, 9498,S, 4236,A, 198,T, 8412,N, 9498,G, 5274,E, 2148 ]




                            It is missing data.




                            The last three are the orange blocks, the second to last three are the black blocks, the third to last three are the green blocks and the first 2 are only half of the yellow blocks, the x and k are missing.




                            However:




                            There are two pairs letters with same numbers, 9498 and 8412.




                            Also:




                            John stated elsewhere its just a numbers game...which makes sense because the letters spell out Stack Exchange







                            share|improve this answer












                            $endgroup$



















                              3
















                              $begingroup$

                              My answer:




                              When arranging the numbers in the order of the word Stack Exchange, and then doing an absolute value of the difference between any number and the following number, I get a pattern that the difference value rises then drops alternatingly, except at the Black A where the value remains the same. (Differences: 4176,8214,8214,3150,5088,1950,5274,2152,4252,0,4224,3126 - rise fall neutral fall rise fall rise fall rise fall rise fall)




                              Below is Meant as comment, but alas



                              @John S posted the following in a comment at https://puzzling.stackexchange.com/a/83526/60039




                              [ C, 8412, E, 174,H, 5246,C, 7398,A, 9498,S, 4236,A, 198,T, 8412,N, 9498,G, 5274,E, 2148 ]




                              It is missing data.




                              The last three are the orange blocks, the second to last three are the black blocks, the third to last three are the green blocks and the first 2 are only half of the yellow blocks, the x and k are missing.




                              However:




                              There are two pairs letters with same numbers, 9498 and 8412.




                              Also:




                              John stated elsewhere its just a numbers game...which makes sense because the letters spell out Stack Exchange







                              share|improve this answer












                              $endgroup$

















                                3














                                3










                                3







                                $begingroup$

                                My answer:




                                When arranging the numbers in the order of the word Stack Exchange, and then doing an absolute value of the difference between any number and the following number, I get a pattern that the difference value rises then drops alternatingly, except at the Black A where the value remains the same. (Differences: 4176,8214,8214,3150,5088,1950,5274,2152,4252,0,4224,3126 - rise fall neutral fall rise fall rise fall rise fall rise fall)




                                Below is Meant as comment, but alas



                                @John S posted the following in a comment at https://puzzling.stackexchange.com/a/83526/60039




                                [ C, 8412, E, 174,H, 5246,C, 7398,A, 9498,S, 4236,A, 198,T, 8412,N, 9498,G, 5274,E, 2148 ]




                                It is missing data.




                                The last three are the orange blocks, the second to last three are the black blocks, the third to last three are the green blocks and the first 2 are only half of the yellow blocks, the x and k are missing.




                                However:




                                There are two pairs letters with same numbers, 9498 and 8412.




                                Also:




                                John stated elsewhere its just a numbers game...which makes sense because the letters spell out Stack Exchange







                                share|improve this answer












                                $endgroup$



                                My answer:




                                When arranging the numbers in the order of the word Stack Exchange, and then doing an absolute value of the difference between any number and the following number, I get a pattern that the difference value rises then drops alternatingly, except at the Black A where the value remains the same. (Differences: 4176,8214,8214,3150,5088,1950,5274,2152,4252,0,4224,3126 - rise fall neutral fall rise fall rise fall rise fall rise fall)




                                Below is Meant as comment, but alas



                                @John S posted the following in a comment at https://puzzling.stackexchange.com/a/83526/60039




                                [ C, 8412, E, 174,H, 5246,C, 7398,A, 9498,S, 4236,A, 198,T, 8412,N, 9498,G, 5274,E, 2148 ]




                                It is missing data.




                                The last three are the orange blocks, the second to last three are the black blocks, the third to last three are the green blocks and the first 2 are only half of the yellow blocks, the x and k are missing.




                                However:




                                There are two pairs letters with same numbers, 9498 and 8412.




                                Also:




                                John stated elsewhere its just a numbers game...which makes sense because the letters spell out Stack Exchange








                                share|improve this answer















                                share|improve this answer




                                share|improve this answer



                                share|improve this answer








                                edited May 2 at 20:39

























                                answered May 2 at 20:16









                                wolfsshieldwolfsshield

                                2367 bronze badges




                                2367 bronze badges
























                                    2
















                                    $begingroup$

                                    I would guess it is the:




                                    Yellow block that has the E and the number 174




                                    Why?




                                    All other group of blocks follow the pattern of increasing/decreasing the number as the letters advance through the alphabet.







                                    share|improve this answer










                                    $endgroup$



















                                      2
















                                      $begingroup$

                                      I would guess it is the:




                                      Yellow block that has the E and the number 174




                                      Why?




                                      All other group of blocks follow the pattern of increasing/decreasing the number as the letters advance through the alphabet.







                                      share|improve this answer










                                      $endgroup$

















                                        2














                                        2










                                        2







                                        $begingroup$

                                        I would guess it is the:




                                        Yellow block that has the E and the number 174




                                        Why?




                                        All other group of blocks follow the pattern of increasing/decreasing the number as the letters advance through the alphabet.







                                        share|improve this answer










                                        $endgroup$



                                        I would guess it is the:




                                        Yellow block that has the E and the number 174




                                        Why?




                                        All other group of blocks follow the pattern of increasing/decreasing the number as the letters advance through the alphabet.








                                        share|improve this answer













                                        share|improve this answer




                                        share|improve this answer



                                        share|improve this answer










                                        answered May 2 at 17:47









                                        Daniel DuqueDaniel Duque

                                        4492 silver badges8 bronze badges




                                        4492 silver badges8 bronze badges
























                                            2
















                                            $begingroup$

                                            Answer:




                                            The Black A




                                            Reason:




                                            It's the only one that has 11 as a factor







                                            share|improve this answer










                                            $endgroup$



















                                              2
















                                              $begingroup$

                                              Answer:




                                              The Black A




                                              Reason:




                                              It's the only one that has 11 as a factor







                                              share|improve this answer










                                              $endgroup$

















                                                2














                                                2










                                                2







                                                $begingroup$

                                                Answer:




                                                The Black A




                                                Reason:




                                                It's the only one that has 11 as a factor







                                                share|improve this answer










                                                $endgroup$



                                                Answer:




                                                The Black A




                                                Reason:




                                                It's the only one that has 11 as a factor








                                                share|improve this answer













                                                share|improve this answer




                                                share|improve this answer



                                                share|improve this answer










                                                answered May 2 at 21:20









                                                wolfsshieldwolfsshield

                                                2367 bronze badges




                                                2367 bronze badges
























                                                    1
















                                                    $begingroup$

                                                    The answer might be:




                                                    The yellow one




                                                    Reasoning:




                                                    All the other blocks have only letters when looked at from a specific direction.







                                                    share|improve this answer










                                                    $endgroup$














                                                    • $begingroup$
                                                      No, sorry. We are looking for an individual block that just doesn't fit the criteria of all the others. - But that was a reasonable guess!
                                                      $endgroup$
                                                      – John S.
                                                      May 2 at 15:40











                                                    • $begingroup$
                                                      HERE IS THE DATA SET FOR THIS PUZZLE... Now, we are back to our regularly scheduled "What doesn't belong in this pattern?" puzzle. [ C, 8412, E, 174,H, 5246,C, 7398,A, 9498,S, 4236,A, 198,T, 8412,N, 9498,G, 5274,E, 2148 ]
                                                      $endgroup$
                                                      – John S.
                                                      May 2 at 19:06











                                                    • $begingroup$
                                                      Delete this question. Not editing it. ANSWER WAS SIMPLE: take first number and last number , combine. Divide by 2. will give you middle number. ONLY ONE block doesn't fit criteria. People had fun trying, you can delete. Thanks.
                                                      $endgroup$
                                                      – John S.
                                                      May 3 at 11:20















                                                    1
















                                                    $begingroup$

                                                    The answer might be:




                                                    The yellow one




                                                    Reasoning:




                                                    All the other blocks have only letters when looked at from a specific direction.







                                                    share|improve this answer










                                                    $endgroup$














                                                    • $begingroup$
                                                      No, sorry. We are looking for an individual block that just doesn't fit the criteria of all the others. - But that was a reasonable guess!
                                                      $endgroup$
                                                      – John S.
                                                      May 2 at 15:40











                                                    • $begingroup$
                                                      HERE IS THE DATA SET FOR THIS PUZZLE... Now, we are back to our regularly scheduled "What doesn't belong in this pattern?" puzzle. [ C, 8412, E, 174,H, 5246,C, 7398,A, 9498,S, 4236,A, 198,T, 8412,N, 9498,G, 5274,E, 2148 ]
                                                      $endgroup$
                                                      – John S.
                                                      May 2 at 19:06











                                                    • $begingroup$
                                                      Delete this question. Not editing it. ANSWER WAS SIMPLE: take first number and last number , combine. Divide by 2. will give you middle number. ONLY ONE block doesn't fit criteria. People had fun trying, you can delete. Thanks.
                                                      $endgroup$
                                                      – John S.
                                                      May 3 at 11:20













                                                    1














                                                    1










                                                    1







                                                    $begingroup$

                                                    The answer might be:




                                                    The yellow one




                                                    Reasoning:




                                                    All the other blocks have only letters when looked at from a specific direction.







                                                    share|improve this answer










                                                    $endgroup$



                                                    The answer might be:




                                                    The yellow one




                                                    Reasoning:




                                                    All the other blocks have only letters when looked at from a specific direction.








                                                    share|improve this answer













                                                    share|improve this answer




                                                    share|improve this answer



                                                    share|improve this answer










                                                    answered May 2 at 15:39









                                                    Krad CigolKrad Cigol

                                                    1,0742 silver badges12 bronze badges




                                                    1,0742 silver badges12 bronze badges














                                                    • $begingroup$
                                                      No, sorry. We are looking for an individual block that just doesn't fit the criteria of all the others. - But that was a reasonable guess!
                                                      $endgroup$
                                                      – John S.
                                                      May 2 at 15:40











                                                    • $begingroup$
                                                      HERE IS THE DATA SET FOR THIS PUZZLE... Now, we are back to our regularly scheduled "What doesn't belong in this pattern?" puzzle. [ C, 8412, E, 174,H, 5246,C, 7398,A, 9498,S, 4236,A, 198,T, 8412,N, 9498,G, 5274,E, 2148 ]
                                                      $endgroup$
                                                      – John S.
                                                      May 2 at 19:06











                                                    • $begingroup$
                                                      Delete this question. Not editing it. ANSWER WAS SIMPLE: take first number and last number , combine. Divide by 2. will give you middle number. ONLY ONE block doesn't fit criteria. People had fun trying, you can delete. Thanks.
                                                      $endgroup$
                                                      – John S.
                                                      May 3 at 11:20
















                                                    • $begingroup$
                                                      No, sorry. We are looking for an individual block that just doesn't fit the criteria of all the others. - But that was a reasonable guess!
                                                      $endgroup$
                                                      – John S.
                                                      May 2 at 15:40











                                                    • $begingroup$
                                                      HERE IS THE DATA SET FOR THIS PUZZLE... Now, we are back to our regularly scheduled "What doesn't belong in this pattern?" puzzle. [ C, 8412, E, 174,H, 5246,C, 7398,A, 9498,S, 4236,A, 198,T, 8412,N, 9498,G, 5274,E, 2148 ]
                                                      $endgroup$
                                                      – John S.
                                                      May 2 at 19:06











                                                    • $begingroup$
                                                      Delete this question. Not editing it. ANSWER WAS SIMPLE: take first number and last number , combine. Divide by 2. will give you middle number. ONLY ONE block doesn't fit criteria. People had fun trying, you can delete. Thanks.
                                                      $endgroup$
                                                      – John S.
                                                      May 3 at 11:20















                                                    $begingroup$
                                                    No, sorry. We are looking for an individual block that just doesn't fit the criteria of all the others. - But that was a reasonable guess!
                                                    $endgroup$
                                                    – John S.
                                                    May 2 at 15:40





                                                    $begingroup$
                                                    No, sorry. We are looking for an individual block that just doesn't fit the criteria of all the others. - But that was a reasonable guess!
                                                    $endgroup$
                                                    – John S.
                                                    May 2 at 15:40













                                                    $begingroup$
                                                    HERE IS THE DATA SET FOR THIS PUZZLE... Now, we are back to our regularly scheduled "What doesn't belong in this pattern?" puzzle. [ C, 8412, E, 174,H, 5246,C, 7398,A, 9498,S, 4236,A, 198,T, 8412,N, 9498,G, 5274,E, 2148 ]
                                                    $endgroup$
                                                    – John S.
                                                    May 2 at 19:06





                                                    $begingroup$
                                                    HERE IS THE DATA SET FOR THIS PUZZLE... Now, we are back to our regularly scheduled "What doesn't belong in this pattern?" puzzle. [ C, 8412, E, 174,H, 5246,C, 7398,A, 9498,S, 4236,A, 198,T, 8412,N, 9498,G, 5274,E, 2148 ]
                                                    $endgroup$
                                                    – John S.
                                                    May 2 at 19:06













                                                    $begingroup$
                                                    Delete this question. Not editing it. ANSWER WAS SIMPLE: take first number and last number , combine. Divide by 2. will give you middle number. ONLY ONE block doesn't fit criteria. People had fun trying, you can delete. Thanks.
                                                    $endgroup$
                                                    – John S.
                                                    May 3 at 11:20




                                                    $begingroup$
                                                    Delete this question. Not editing it. ANSWER WAS SIMPLE: take first number and last number , combine. Divide by 2. will give you middle number. ONLY ONE block doesn't fit criteria. People had fun trying, you can delete. Thanks.
                                                    $endgroup$
                                                    – John S.
                                                    May 3 at 11:20











                                                    1
















                                                    $begingroup$

                                                    I noticed that




                                                    if you open out the net of each, centred on the side showing the letter, any numbers above, below and to the right are all aligned in the same direction as the letter; and the number on the left is on its side, with its top pointing to the left. The only exception is the Yellow K block, where the number on the right isn't aligned in the same direction as the letter.







                                                    share|improve this answer










                                                    $endgroup$










                                                    • 1




                                                      $begingroup$
                                                      Looks like there is more than one valid solution :(
                                                      $endgroup$
                                                      – MichaelMaggs
                                                      May 2 at 16:51






                                                    • 5




                                                      $begingroup$
                                                      If there's more than one block that's different from the others in a notable way and the challenge is to figure out what specific difference you had in mind, then the puzzle is basically "read the puzzle author's mind". Let's hope it turns out that the odd-one-out property John S intends is much more compelling than the two other perfectly plausible ones suggested so far...
                                                      $endgroup$
                                                      – Gareth McCaughan
                                                      May 2 at 17:42






                                                    • 1




                                                      $begingroup$
                                                      It is the other way around >! This is a hint.
                                                      $endgroup$
                                                      – Daniel Duque
                                                      May 2 at 18:33






                                                    • 1




                                                      $begingroup$
                                                      @JohnS. typically we use Rot13 to hide hints or spoilers in comments. But I’d strongly recommend you put the hints in the original post, since they would be considered part of the overall puzzle
                                                      $endgroup$
                                                      – PiIsNot3
                                                      May 2 at 19:12







                                                    • 1




                                                      $begingroup$
                                                      @JohnS. A "What's next?" like you describe should also be closed for being too broad...
                                                      $endgroup$
                                                      – noedne
                                                      May 2 at 21:24















                                                    1
















                                                    $begingroup$

                                                    I noticed that




                                                    if you open out the net of each, centred on the side showing the letter, any numbers above, below and to the right are all aligned in the same direction as the letter; and the number on the left is on its side, with its top pointing to the left. The only exception is the Yellow K block, where the number on the right isn't aligned in the same direction as the letter.







                                                    share|improve this answer










                                                    $endgroup$










                                                    • 1




                                                      $begingroup$
                                                      Looks like there is more than one valid solution :(
                                                      $endgroup$
                                                      – MichaelMaggs
                                                      May 2 at 16:51






                                                    • 5




                                                      $begingroup$
                                                      If there's more than one block that's different from the others in a notable way and the challenge is to figure out what specific difference you had in mind, then the puzzle is basically "read the puzzle author's mind". Let's hope it turns out that the odd-one-out property John S intends is much more compelling than the two other perfectly plausible ones suggested so far...
                                                      $endgroup$
                                                      – Gareth McCaughan
                                                      May 2 at 17:42






                                                    • 1




                                                      $begingroup$
                                                      It is the other way around >! This is a hint.
                                                      $endgroup$
                                                      – Daniel Duque
                                                      May 2 at 18:33






                                                    • 1




                                                      $begingroup$
                                                      @JohnS. typically we use Rot13 to hide hints or spoilers in comments. But I’d strongly recommend you put the hints in the original post, since they would be considered part of the overall puzzle
                                                      $endgroup$
                                                      – PiIsNot3
                                                      May 2 at 19:12







                                                    • 1




                                                      $begingroup$
                                                      @JohnS. A "What's next?" like you describe should also be closed for being too broad...
                                                      $endgroup$
                                                      – noedne
                                                      May 2 at 21:24













                                                    1














                                                    1










                                                    1







                                                    $begingroup$

                                                    I noticed that




                                                    if you open out the net of each, centred on the side showing the letter, any numbers above, below and to the right are all aligned in the same direction as the letter; and the number on the left is on its side, with its top pointing to the left. The only exception is the Yellow K block, where the number on the right isn't aligned in the same direction as the letter.







                                                    share|improve this answer










                                                    $endgroup$



                                                    I noticed that




                                                    if you open out the net of each, centred on the side showing the letter, any numbers above, below and to the right are all aligned in the same direction as the letter; and the number on the left is on its side, with its top pointing to the left. The only exception is the Yellow K block, where the number on the right isn't aligned in the same direction as the letter.








                                                    share|improve this answer













                                                    share|improve this answer




                                                    share|improve this answer



                                                    share|improve this answer










                                                    answered May 2 at 16:39









                                                    MichaelMaggsMichaelMaggs

                                                    8692 silver badges16 bronze badges




                                                    8692 silver badges16 bronze badges










                                                    • 1




                                                      $begingroup$
                                                      Looks like there is more than one valid solution :(
                                                      $endgroup$
                                                      – MichaelMaggs
                                                      May 2 at 16:51






                                                    • 5




                                                      $begingroup$
                                                      If there's more than one block that's different from the others in a notable way and the challenge is to figure out what specific difference you had in mind, then the puzzle is basically "read the puzzle author's mind". Let's hope it turns out that the odd-one-out property John S intends is much more compelling than the two other perfectly plausible ones suggested so far...
                                                      $endgroup$
                                                      – Gareth McCaughan
                                                      May 2 at 17:42






                                                    • 1




                                                      $begingroup$
                                                      It is the other way around >! This is a hint.
                                                      $endgroup$
                                                      – Daniel Duque
                                                      May 2 at 18:33






                                                    • 1




                                                      $begingroup$
                                                      @JohnS. typically we use Rot13 to hide hints or spoilers in comments. But I’d strongly recommend you put the hints in the original post, since they would be considered part of the overall puzzle
                                                      $endgroup$
                                                      – PiIsNot3
                                                      May 2 at 19:12







                                                    • 1




                                                      $begingroup$
                                                      @JohnS. A "What's next?" like you describe should also be closed for being too broad...
                                                      $endgroup$
                                                      – noedne
                                                      May 2 at 21:24












                                                    • 1




                                                      $begingroup$
                                                      Looks like there is more than one valid solution :(
                                                      $endgroup$
                                                      – MichaelMaggs
                                                      May 2 at 16:51






                                                    • 5




                                                      $begingroup$
                                                      If there's more than one block that's different from the others in a notable way and the challenge is to figure out what specific difference you had in mind, then the puzzle is basically "read the puzzle author's mind". Let's hope it turns out that the odd-one-out property John S intends is much more compelling than the two other perfectly plausible ones suggested so far...
                                                      $endgroup$
                                                      – Gareth McCaughan
                                                      May 2 at 17:42






                                                    • 1




                                                      $begingroup$
                                                      It is the other way around >! This is a hint.
                                                      $endgroup$
                                                      – Daniel Duque
                                                      May 2 at 18:33






                                                    • 1




                                                      $begingroup$
                                                      @JohnS. typically we use Rot13 to hide hints or spoilers in comments. But I’d strongly recommend you put the hints in the original post, since they would be considered part of the overall puzzle
                                                      $endgroup$
                                                      – PiIsNot3
                                                      May 2 at 19:12







                                                    • 1




                                                      $begingroup$
                                                      @JohnS. A "What's next?" like you describe should also be closed for being too broad...
                                                      $endgroup$
                                                      – noedne
                                                      May 2 at 21:24







                                                    1




                                                    1




                                                    $begingroup$
                                                    Looks like there is more than one valid solution :(
                                                    $endgroup$
                                                    – MichaelMaggs
                                                    May 2 at 16:51




                                                    $begingroup$
                                                    Looks like there is more than one valid solution :(
                                                    $endgroup$
                                                    – MichaelMaggs
                                                    May 2 at 16:51




                                                    5




                                                    5




                                                    $begingroup$
                                                    If there's more than one block that's different from the others in a notable way and the challenge is to figure out what specific difference you had in mind, then the puzzle is basically "read the puzzle author's mind". Let's hope it turns out that the odd-one-out property John S intends is much more compelling than the two other perfectly plausible ones suggested so far...
                                                    $endgroup$
                                                    – Gareth McCaughan
                                                    May 2 at 17:42




                                                    $begingroup$
                                                    If there's more than one block that's different from the others in a notable way and the challenge is to figure out what specific difference you had in mind, then the puzzle is basically "read the puzzle author's mind". Let's hope it turns out that the odd-one-out property John S intends is much more compelling than the two other perfectly plausible ones suggested so far...
                                                    $endgroup$
                                                    – Gareth McCaughan
                                                    May 2 at 17:42




                                                    1




                                                    1




                                                    $begingroup$
                                                    It is the other way around >! This is a hint.
                                                    $endgroup$
                                                    – Daniel Duque
                                                    May 2 at 18:33




                                                    $begingroup$
                                                    It is the other way around >! This is a hint.
                                                    $endgroup$
                                                    – Daniel Duque
                                                    May 2 at 18:33




                                                    1




                                                    1




                                                    $begingroup$
                                                    @JohnS. typically we use Rot13 to hide hints or spoilers in comments. But I’d strongly recommend you put the hints in the original post, since they would be considered part of the overall puzzle
                                                    $endgroup$
                                                    – PiIsNot3
                                                    May 2 at 19:12





                                                    $begingroup$
                                                    @JohnS. typically we use Rot13 to hide hints or spoilers in comments. But I’d strongly recommend you put the hints in the original post, since they would be considered part of the overall puzzle
                                                    $endgroup$
                                                    – PiIsNot3
                                                    May 2 at 19:12





                                                    1




                                                    1




                                                    $begingroup$
                                                    @JohnS. A "What's next?" like you describe should also be closed for being too broad...
                                                    $endgroup$
                                                    – noedne
                                                    May 2 at 21:24




                                                    $begingroup$
                                                    @JohnS. A "What's next?" like you describe should also be closed for being too broad...
                                                    $endgroup$
                                                    – noedne
                                                    May 2 at 21:24



                                                    Popular posts from this blog

                                                    Tamil (spriik) Luke uk diar | Nawigatjuun

                                                    Align equal signs while including text over equalitiesAMS align: left aligned text/math plus multicolumn alignmentMultiple alignmentsAligning equations in multiple placesNumbering and aligning an equation with multiple columnsHow to align one equation with another multline equationUsing \ in environments inside the begintabularxNumber equations and preserving alignment of equal signsHow can I align equations to the left and to the right?Double equation alignment problem within align enviromentAligned within align: Why are they right-aligned?

                                                    Where does the image of a data connector as a sharp metal spike originate from?Where does the concept of infected people turning into zombies only after death originate from?Where does the motif of a reanimated human head originate?Where did the notion that Dragons could speak originate?Where does the archetypal image of the 'Grey' alien come from?Where did the suffix '-Man' originate?Where does the notion of being injured or killed by an illusion originate?Where did the term “sophont” originate?Where does the trope of magic spells being driven by advanced technology originate from?Where did the term “the living impaired” originate?